An Atwoods Machine hanging from the ceiling

  • Thread starter x86
  • Start date
  • Tags
    Machine
In summary: So how does the tension in A apply to the masses?The tension in A acts on to the 2 objects (and rope) together, as if they were 1 object. However, since the rope is not directly attached to either mass, the tension in A only applies to the mass that is attached to the rope.
  • #1
256
18

Homework Statement


This isn't a homework problem, but I thought it would fit best in this section because it is an actual problem. Therefore, I don't know what the solution is, but it interests me.

First let me start off by saying I've solved Atwoods machines before but now that we've introduced a rope A that the Atwoods machine is hanging from, it gets very confusing for me.

The question that I'd like to figure out is:
What is the tension in rope A if the Atwoods machine is at rest on the ceiling, but the masses are not? m2 is heavier than m1. The rope is mass less and has no friction.

atwoodmachineceiling.jpg




Homework Equations


Fnet = ma
w = mg
g = |9.8 m/s^2|



The Attempt at a Solution



I'm first off going to start with what I know.

T1 = T2 = T for m1 and m2

The force acting on m1
Fnet = T - m1a1

The force acting on the mass m2
Fnet = T - m2a2

Up is the positive direction, and m2 is going down, so our new equations are

Fnet = T - m1a
Fnet = m2a - T

But this is sort of where I get confused, because the Atwoods machine is hanging from the ceiling.

I know that m2 has a tension force acting on it in the up direction, and the rope has a resultant force from this tension which is pulling the string down- this causes mass m1 to have a Force in the up direction and its resultant force is what causes m2's force in the up direction.

So the rope has no net force acting on it, because both of the tensions from m1 and m2 on the rope cancel out.

Now if we have a rope A that is applying a tension in the up direction to our rope.

So our rope has a force pulling it upwards.

Since the rope is mass less we have to "skip" it and pretend its not there and apply this tension of force A onto both of the masses.

But I have no idea how this tension applies.

Do I have to divide it in half because I have two masses?

And if I did do that, it would change the acceleration of the two masses, but that doesn't make sense to me- because if we just pretended the Atwoods machine was hanging from a ceiling and ignored the tension A, then the acceleration would be different from what I would get if I didn't ignore it. And I know this is wrong.
 
Physics news on Phys.org
  • #2
x86 said:
So the rope has no net force acting on it, because both of the tensions from m1 and m2 on the rope cancel out.

True that those forces cancel out. False that it has no net force acting on it. What about the weight of the objects, is that not a force acting on the rope?
 
  • #3
x86 said:
Since the rope is mass less we have to "skip" it and pretend its not there and apply this tension of force A onto both of the masses.

But I have no idea how this tension applies.

Do I have to divide it in half because I have two masses?

The tension from A acts on to the 2 objects (and rope) together, as if they were 1 object
 
  • #4
Nathanael said:
True that those forces cancel out. False that it has no net force acting on it. What about the weight of the objects, is that not a force acting on the rope?

Doesn't the weight not act on the rope? The reactant force of the weight is the force of the mass on the earth.

Nathanael said:
The tension from A acts on to the 2 objects (and rope) together, as if they were 1 object

If it is (m1+m2)g then doesn't that basically cancel the force of gravity acting on the masses, thus making the whole system not move at all?
 
  • #5
x86 said:
Doesn't the weight not act on the rope? The reactant force of the weight is the force of the mass on the earth.
Yes that is the reactant force, but the weight is still attatched to the rope, and therefore exerts a force on the rope.

If I pull down on one of the objects, (with my hand,) the reactant force will be that the object pulls up on my hand. But there will still be a force on the rope, right?

Basically, since the rope is attatched to the object, it's essentially a part of the object, so the force of gravity on the object is also a force on the rope.


x86 said:
If it is (m1+m2)g then doesn't that basically cancel the force of gravity acting on the masses, thus making the whole system not move at all?

It does cancel the force of gravity of the system, thus keeping the whole system from falling down. But it only acts on the entire system as a whole.

But rope A is not directly attached to the other rope/weights. If it were, then the objects would not move at all like you said. Rope A is holding up the pulley-thing, so it keeps it (and the rope/weights on it) from falling down, but the rope/weights still have freedom to slide over it.

(The reason I kept saying "rope/weights" is because it's essentially a single object)
 
  • Like
Likes 1 person
  • #6
Nathanael said:
Yes that is the reactant force, but the weight is still attatched to the rope, and therefore exerts a force on the rope.

If I pull down on one of the objects, (with my hand,) the reactant force will be that the object pulls up on my hand. But there will still be a force on the rope, right?

Basically, since the rope is attatched to the object, it's essentially a part of the object, so the force of gravity on the object is also a force on the rope.




It does cancel the force of gravity of the system, thus keeping the whole system from falling down. But it only acts on the entire system as a whole.

But rope A is not directly attached to the other rope/weights. If it were, then the objects would not move at all like you said. Rope A is holding up the pulley-thing, so it keeps it (and the rope/weights on it) from falling down, but the rope/weights still have freedom to slide over it.

(The reason I kept saying "rope/weights" is because it's essentially a single object)

Yes, that is true. But for the first part of your post, if the force of gravity for a mass, say m1, acts on the rope and pulls it down, doesn't that essentially cause m2 to receive an upward force of m1g plus an upward force of T?

Therefore the force on say m2 would now be

Fnet = T + m1g - m2g


That means that we would receive a different answer for the accelerations.
 
Last edited:
  • #7
But the mass pulling it down is what causes the tension, so it's already accounted for in the tension term
 
  • Like
Likes 1 person
  • #8
Nathanael said:
But the mass pulling it down is what causes the tension, so it's already accounted for in the tension term

True. But aren't both of those forces canceled out? There are two tensions, each in opposite directions. Therefore the net force on the rope is 0. How does that effect the rope A?

You said in your prior post it cancels out, but then what causes the pulley to have a Force downwards?

Also, A = 2T basically?
 
  • #9
x86 said:
True. But aren't both of those forces canceled out? There are two tensions, each in opposite directions. Therefore the net force on the rope is 0.
The tension forces do cancel out, so if you're looking at the net force on the entire system, tension is irrelevant (because it cancels out) so you just have [itex]F_{system.net}=g(m_2-m_1)[/itex]

But that doesn't necessarily mean that the net force on the rope is zero, because tension is not the only force involved in the situation.

(If you're looking at a single block, though, instead of the entire system, then tension becomes relevant again, because it no longer cancels out.)
x86 said:
Also, A = 2T basically?
No, that wouldn't be true. Tension is just with how much force the rope is being pulled apart (or, another way of looking at it is, with how much force the rope is trying to pull itself back)

The tension in the lower rope doesn't have any special relationship with the tension in rope A

x86 said:
You said in your prior post it cancels out, but then what causes the pulley to have a Force downwards?
I'm sorry, I don't quite understand what you are saying. Can you elaborate?
 
Last edited:
  • Like
Likes 1 person
  • #10
Nathanael said:
The tension forces do cancel out, so if you're looking at the net force on the entire system, tension is irrelevant (because it cancels out) so you just have [itex]F_{system.net}=g(m_2-m_1)[/itex]

But that doesn't necessarily mean that the net force on the rope is zero, because tension is not the only force involved in the situation.

(If you're looking at a single block, though, instead of the entire system, then tension becomes relevant again, because it no longer cancels out.)

No, that wouldn't be true. Tension is just with how much force the rope is being pulled apart (or, another way of looking at it is, with how much force the rope is trying to pull itself back)

The tension in the lower rope doesn't have any special relationship with the tension in rope AI'm sorry, I don't quite understand what you are saying. Can you elaborate?

Okay, thanks. I think I understand it now. But isn't the net force g(m1 + m2) and not g(m2 - m1)?
 
  • #11
x86 said:
Okay, thanks. I think I understand it now. But isn't the net force g(m1 + m2) and not g(m2 - m1)?

Well they're in opposite directions, that's why I subtracted.(In a sense, they're in the same direction. But the reason why it's actually in opposite directions is because one is trying to move the rope "to the right" and the other is trying to move the rope "to the left")
 
  • #12
x86 said:
Okay, thanks. I think I understand it now. But isn't the net force g(m1 + m2) and not g(m2 - m1)?

Are you talking about the net force on the system comprising the two blocks and the pulley ? If yes , net force is neither g(m1 + m2) nor g(m2 - m1) .
 
  • #13
Tanya Sharma said:
Are you talking about the net force on the system comprising the two blocks and the pulley ? If yes , net force is neither g(m1 + m2) nor g(m2 - m1) .

What would the net force on the rope and weights be? We're talking about a simplified situation where the pulley only serves the purpose of changing the direction.
 
  • #14
Nathanael said:
What would the net force on the rope and weights be? We're talking about a simplified situation where the pulley only serves the purpose of changing the direction.

(m1+m2)g - 2T
 
  • #15
x86 said:
Okay, thanks. I think I understand it now. But isn't the net force g(m1 + m2) and not g(m2 - m1)?

Since the objects are connected by a rope, which travels over a pulley, both WEIGHT forces act down. The weight force from one block exerts a force on the rope which exerts a force on the other block, therefore the system is comparable to something like this (see attached) ignoring the mass of the pulley of course. What's the net force on this system?
 

Attachments

  • atwoods analogy.png
    atwoods analogy.png
    3.7 KB · Views: 476
  • #16
The system comprising of the 2 blocks and rope of course.
 
  • #17
Tanya Sharma said:
(m1+m2)g - 2T

Ok yes I suppose that would be the net force on the center of mass. So we weren't technically talking about the entire system

But in a sense we were talking about the whole system, just over a different path (the pulley changes the direction of one of the mg's and one T leaving you (m1-m2)g)

I don't know what this "net force" would be technically called
 
Last edited:
  • #18
Nathanael said:
But in a sense we were talking about the whole system, just over a different path (the pulley changes the direction of one of the mg's and one T leaving you (m1-m2)g)

Even if you consider only the blocks ,then too (m1-m2)g doesn't make any sense .

Whichever system you want to consider , only one block ,both the blocks ,both the blocks + pulley ; (m1-m2)g is not the net force on any of them .
 
  • #19
The system is 2 blocks and a rope.

Do you really not understand where I'm coming from at all? (m2-m1)g doesn't make ANY sense?

The path it moves is an upside down U. Imagine you could somehow "bend space" to make the path straight. Then you would have the picture that BiGyElLoWhAt attatched a few posts ago, with both of the tensions cancelling out and the net force being (m2-m1)g


Sure, the net force on the Center-of-Mass would be (m2+m1)g-2T but what I am talking about is the Net force "along the path of motion" (so to speak).
 
  • #20
Nathanael said:
The system is 2 blocks and a rope.

The path it moves is an upside down U. Imagine you could somehow "bend space" to make the path straight. Then you would have the picture that BiGyElLoWhAt attatched a few posts ago, with both of the tensions cancelling out and the net force being (m2-m1)g

Sure, the net force on the Center-of-Mass would be (m2+m1)g-2T but what I am talking about is the Net force "along the path of motion" (so to speak).

So you believe that net force (m2-m1)g acts on the system of 2 blocks and rope . Then what is the acceleration of the COM of the system (2 blocks and rope) ?
 
  • #21
Tanya Sharma said:
So you believe that net force (m2-m1)g acts on the system of 2 blocks and rope . Then what is the acceleration of the COM of the system (2 blocks and rope) ?

I've already said several times that the equation you gave desribes the acceleration of the COM of the system

[itex]a_{com}=\frac{g(m_1+m_2)-2T}{m_1+m_2}[/itex]


My point is that we're not talking about the COM...
 
  • #22
Nathanael said:
My point is that we're not talking about the COM...

In that case don't term (m2-m1)g as the net force . It is not the force along the inverted U path you have been talking about .
 
  • #23
Tanya Sharma said:
In that case don't term (m2-m1)g as the net force . It is not the force along the inverted U path you have been talking about .

"Net force" is the only way I could think of to say it to the OP (we were talking about the total force (that's why I said "net force") as opposed to the force on a single block)

I DO understand it would not technically be the net force, (but the OP was not thinking about the of center of mass)

But what would you call it? (I'm still inclined to call it a "net force" because I don't know what else to call it)
 
  • #24
Nathanael said:
True that those forces cancel out. False that it has no net force acting on it. What about the weight of the objects, is that not a force acting on the rope?

The net force acting on the rope is zero as it is massless .The weight of the object is not a force acting on the rope.

Nathanael said:
It does cancel the force of gravity of the system, thus keeping the whole system from falling down. But it only acts on the entire system as a whole.

The tension in the upper string doesn't cancel the force due to gravity of the system .There is a net downward force on the system.
 
  • #25
Nathanael said:
But the mass pulling it down is what causes the tension, so it's already accounted for in the tension term

The tension T and the force due to gravity are two separate forces .You need to account for both.The difference between the two is what accelerates the block .

Nathanael said:
The tension forces do cancel out, so if you're looking at the net force on the entire system, tension is irrelevant (because it cancels out) so you just have [itex]F_{system.net}=g(m_2-m_1)[/itex]

Again ,this is incorrect.

Nathanael said:
But that doesn't necessarily mean that the net force on the rope is zero, because tension is not the only force involved in the situation.

No .The net force on the rope is zero .

Nathanael said:
The tension in the lower rope doesn't have any special relationship with the tension in rope A

It does have a relationship .Assuming the pulley is massless ,the tension in the upper string is twice the tension in the lower string.
 
  • #26
Tanya Sharma said:
The net force acting on the rope is zero as it is massless .The weight of the object is not a force acting on the rope.

He asked why the rope moves if there's no force on it, meaning that he was looking at the rope as an object with mass...


There's no reason to be so pedantic when it's irrelevant to his confusion
 
  • #27
Nathanael said:
He asked why the rope moves if there's no force on it, meaning that he was looking at the rope as an object with mass...

A massless rope doesn't require a force to accelerate it .

Nathanael said:
There's no reason to be so pedantic when it's irrelevant to his confusion

I am not being pedantic .What I have discussed is quite relevant to the original problem.The information you provided to OP is mostly incorrect . The OP's main concern wasn't addressed properly .His confusion can be addressed by utilising the FBD's of the two blocks as well as the pulley.

From the OP :

x86 said:
The question that I'd like to figure out is:
What is the tension in rope A if the Atwoods machine is at rest on the ceiling, but the masses are not? m2 is heavier than m1. The rope is mass less and has no friction.
 
Last edited:
  • #28
If the pulley is massless then it doesn't exert a torque on the system of the rope and 2 masses.
##F_{net_{m_{1}}}= T_{1} - m_{1}g##
##F_{net_{m_{2}}}= T_{2} - m_{2}g##
##F_{net_{rope+m1+m2}}= T_{1} + T_{2} - g(m_{1}+m_{2})## And with no torques we have
##F_{net_{r+1+2}} = 2T - g(m_{1}+m_{2})##

Since the tensions are the same.
 
  • #29
To x86, Sorry.

It seems I've caused more problems then I've solved. You asked me if A=2T and I told you no without even giving it a worthy analysis. (It's clear from the COM-perspective that A does = 2T) I'm sorry for any misconceptions I've promoted. I realize now that my intuition isn't trustworthy in the slightest, and that I was too naive to analyze my first instincts.
So I want to apologize. At least we've now both learned from this thread.
 
  • #30
Nathanael said:
To x86, Sorry.

It seems I've caused more problems then I've solved. You asked me if A=2T and I told you no without even giving it a worthy analysis. (It's clear from the COM-perspective that A does = 2T) I'm sorry for any misconceptions I've promoted. I realize now that my intuition isn't trustworthy in the slightest, and that I was too naive to analyze my first instincts.
So I want to apologize. At least we've now both learned from this thread.

It's fine. Physics can be confusing at times, I guess.In general,The thing that confuses me, is when people think of Atwood machines as 2D objects in the -x and +x axis

Then the tension cancels out, and the rope has a net force of 0. But shouldn't it also have a force in the up direction, to stop it from going down? (the tension resultant forces from the masses)

In a sense, the rope connecting the two masses has three forces acting on it, right?

Two downward tension forces equal to T, and one force directed up equal to 2T. Thus the mass less rope has a net force of zero.
 
Last edited:

1. What is an Atwoods Machine hanging from the ceiling?

An Atwoods Machine is a physics apparatus used to study the principles of mechanics, specifically the laws of motion and the concept of equilibrium. It consists of two masses connected by a string or rope that passes over a pulley. When one mass is released, it will accelerate downward due to the force of gravity, causing the other mass to accelerate upward.

2. How does an Atwoods Machine hanging from the ceiling work?

The Atwoods Machine demonstrates the principles of Newton's second law of motion, which states that the acceleration of an object is directly proportional to the net force acting on it and inversely proportional to its mass. In this case, the two masses are connected by a string, and the force of gravity acts on both of them. As one mass accelerates downward, the other mass will accelerate upward due to the tension in the string.

3. What are the applications of an Atwoods Machine hanging from the ceiling?

An Atwoods Machine is used in physics education to teach students about the principles of mechanics and to conduct experiments related to force, acceleration, and equilibrium. It can also be used to demonstrate concepts such as friction, energy conservation, and rotational motion.

4. How is an Atwoods Machine hanging from the ceiling set up?

To set up an Atwoods Machine, two masses of different weights are connected by a string or rope that passes over a pulley. The pulley should be securely attached to the ceiling, and the string should be taut. The masses should be able to move freely without any obstructions. The machine can be adjusted by changing the masses, the length of the string, or the angle of the pulley.

5. What are some potential sources of error when using an Atwoods Machine hanging from the ceiling?

Some potential sources of error when using an Atwoods Machine include friction in the pulley, air resistance, and the weight of the string itself. These factors can affect the accuracy of the measurements and calculations. It is important to minimize these sources of error by using smooth pulleys, conducting experiments in a vacuum, and using lightweight strings.

Similar threads

  • Introductory Physics Homework Help
2
Replies
38
Views
1K
  • Introductory Physics Homework Help
Replies
14
Views
958
  • Introductory Physics Homework Help
Replies
3
Views
2K
  • Introductory Physics Homework Help
Replies
17
Views
1K
  • Introductory Physics Homework Help
Replies
4
Views
1K
  • Introductory Physics Homework Help
Replies
16
Views
1K
  • Introductory Physics Homework Help
Replies
2
Views
822
  • Introductory Physics Homework Help
Replies
3
Views
1K
  • Introductory Physics Homework Help
Replies
5
Views
4K
  • Introductory Physics Homework Help
Replies
16
Views
1K
Back
Top